www.vorkurse.de
Ein Projekt von vorhilfe.de
Die Online-Kurse der Vorhilfe

E-Learning leicht gemacht.
Hallo Gast!einloggen | registrieren ]
Startseite · Mitglieder · Teams · Forum · Wissen · Kurse · Impressum
Forenbaum
^ Forenbaum
Status Mathe-Vorkurse
  Status Organisatorisches
  Status Schule
    Status Wiederholung Algebra
    Status Einführung Analysis
    Status Einführung Analytisc
    Status VK 21: Mathematik 6.
    Status VK 37: Kurvendiskussionen
    Status VK Abivorbereitungen
  Status Universität
    Status Lerngruppe LinAlg
    Status VK 13 Analysis I FH
    Status Algebra 2006
    Status VK 22: Algebra 2007
    Status GruMiHH 06
    Status VK 58: Algebra 1
    Status VK 59: Lineare Algebra
    Status VK 60: Analysis
    Status Wahrscheinlichkeitst

Gezeigt werden alle Foren bis zur Tiefe 2

Navigation
 Startseite...
 Neuerdings beta neu
 Forum...
 vorwissen...
 vorkurse...
 Werkzeuge...
 Nachhilfevermittlung beta...
 Online-Spiele beta
 Suchen
 Verein...
 Impressum
Das Projekt
Server und Internetanbindung werden durch Spenden finanziert.
Organisiert wird das Projekt von unserem Koordinatorenteam.
Hunderte Mitglieder helfen ehrenamtlich in unseren moderierten Foren.
Anbieter der Seite ist der gemeinnützige Verein "Vorhilfe.de e.V.".
Partnerseiten
Weitere Fächer:

Open Source FunktionenplotterFunkyPlot: Kostenloser und quelloffener Funktionenplotter für Linux und andere Betriebssysteme
Forum "Folgen und Reihen" - Konvergenz
Konvergenz < Folgen und Reihen < eindimensional < reell < Analysis < Hochschule < Mathe < Vorhilfe
Ansicht: [ geschachtelt ] | ^ Forum "Folgen und Reihen"  | ^^ Alle Foren  | ^ Forenbaum  | Materialien

Konvergenz: Konvergenz?
Status: (Frage) beantwortet Status 
Datum: 22:46 Do 16.10.2008
Autor: meldrolon

Aufgabe
[mm] \summe_{k=1}^{\infty}\bruch{(-1)^{k+1}}{k} [/mm]

Konvergent oder Divergent?

Wenn man die beiden Ausdrücke einzeln betrachtet

[mm] \summe_{k=1}^{\infty}\bruch{1}{k} [/mm]


[mm] \summe_{k=1}^{\infty}(-1)^{k+1} [/mm]

ist ja der erste divergent (harmonische Reihe)

und der zweite ist ja alternierend!?^^

(wieso ist dann
[mm] \summe_{k=1}^{\infty}\bruch{(-1)^{k+1}}{k} [/mm]
konvergent)?

Wenn ich jetzt die ersten paar Werte einsetzte geht die Lösung gegen 0,6..  und es ist ersichtlich das konvergenz vorliegt.  Ich weis dass der Grenzwert ln 2 also ca 0,69 ist aber wie kann man denn zeigen dass der Grenzwert genau ln2 ist?

        
Bezug
Konvergenz: Antwort
Status: (Antwort) fertig Status 
Datum: 23:08 Do 16.10.2008
Autor: schachuzipus

Hallo meldrolon,

> [mm]\summe_{k=1}^{\infty}\bruch{(-1)^{k+1}}{k}[/mm]
>  
> Konvergent oder Divergent?
>  Wenn man die beiden Ausdrücke einzeln betrachtet
>  
> [mm]\summe_{k=1}^{\infty}\bruch{1}{k}[/mm]
>  
>
> [mm]\summe_{k=1}^{\infty}(-1)^{k+1}[/mm]
>  
> ist ja der erste divergent (harmonische Reihe)
>  
> und der zweite ist ja alternierend!?^^

Wie hast du das denn zerlegt?

Das klappt so nicht ... Wie fügst du es mit diesen beiden Teilreihen wieder zusammen zur Ausgangsreihe?


>
> (wieso ist dann [mm]\summe_{k=1}^{\infty}\bruch{(-1)^{k+1}}{k}[/mm]
>  konvergent)?

Ich hoffe, du hast schon einige Konvergenzkriterien kennengelernt ;-)

Deine Reihe [mm] $\summe_{k=1}^{\infty}\bruch{(-1)^{k+1}}{k}$ [/mm] kannst du mit dem []Leibnizkriterium auf Konvergenz untersuchen

>  
> Wenn ich jetzt die ersten paar Werte einsetzte geht die
> Lösung gegen 0,6..  und es ist ersichtlich das konvergenz
> vorliegt.  Ich weis dass der Grenzwert ln 2 also ca 0,69
> ist aber wie kann man denn zeigen dass der Grenzwert genau
> ln2 ist?

Das kannst du durch Entwicklung des [mm] $\ln$ [/mm] in eine Taylorreihe berechnen ...

[mm] $\ln(1+x)=\sum\limits_{k=1}^{\infty}\frac{(-1)^{k+1}}{k}\cdot{}x^k$ [/mm] für [mm] $-1
Dann ist [mm] $\ln(2)=\ln(1+1)=...$ [/mm]


LG

schachuzipus



Bezug
                
Bezug
Konvergenz: Abel'scher GWS
Status: (Mitteilung) Reaktion unnötig Status 
Datum: 11:05 Fr 17.10.2008
Autor: XPatrickX


> Das kannst du durch Entwicklung des [mm]\ln[/mm] in eine Taylorreihe
> berechnen ...
>  
> [mm]\ln(1+x)=\sum\limits_{k=1}^{\infty}\frac{(-1)^{k+1}}{k}\cdot{}x^k[/mm]
> für [mm]-1
>  
> Dann ist [mm]\ln(2)=\ln(1+1)=...[/mm]
>  
>

Wobei man hier noch den Abel'schen Grenzwertsatz berücksichten muss. Der Konvergenzradius ist [mm] |x|\red{<}1, [/mm] sodass für x=1 gar nicht klar ist, dass die Reihe auch wirklich konvergiert.

Gruß Patrick

Bezug
Ansicht: [ geschachtelt ] | ^ Forum "Folgen und Reihen"  | ^^ Alle Foren  | ^ Forenbaum  | Materialien


^ Seitenanfang ^
www.vorkurse.de
[ Startseite | Mitglieder | Teams | Forum | Wissen | Kurse | Impressum ]